Johnny asks you to guess the specific coins he has in his pocket if he has $3.70 in change. You say you need a little
more information, to guess. So he tells you all he has is quarters and dimes and that he has 22 colns in all.

Answers

Answer 1

Answer:

10 quarters and 12 dimes :)

Step-by-step explanation:


Related Questions

Solve for x. Round to the nearest tenth, if necessary.

Answers

Answer:

11.3

Step-by-step explanation:

first we find angle F.

remember, all angles in a triangle always sum up to 180 degrees.

so,

F = 180 - 90 - 61 = 29 degrees

now we use the law of sines.

EF/sin(D) = ED/sin(F) = DF/sin(E)

DF = x

sin(E) = sin(90) = 1

5.5/sin(29) = x/1 = x

x = 11.3

The sum of three consecutive odd numbers is 63. What are the numbers?​

Answers

19,21,23. Adding all of these consecutive odd numbers is equal to 63

Answer:

19, 21 and 23

Step-by-step explanation:

→ Make an algebraic expression for the 3 consecutive numbers

2x + 1, 2x + 3 and 2x + 5

→ Add the expressions together and make it equal to 63

2x + 1 + 2x + 3 + 2x + 5 = 63

→ Simplify

6x + 9 = 63

→ Minus 9 from both sides

6x = 54

→ Divide both sides by 6

x = 9

→ Resubstitute x = 9 into the 3 expressions

x = 9 into 2x + 1 is 19

x = 9 into 2x + 3 is 21

x = 9 into 2x + 5 is 23

For the function G defined by G(x)=5x+3, find G(r+5)

Answers

Given function:

g(x) = 5x + 3

Find

g(r+5)

Substitute x with r = 5:

g(r + 5) = 5(r + 5) + 3 = 5r + 25 + 3 = 5r + 28

Answer:

G ( r + 5 ) = 5r + 28

Step-by-step explanation:

Given ;

G ( x ) = 5x + 3

To Find :-

G ( r + 5 )

Solution :-

plug r + 5 as x in the function.

G ( r + 5 ) = 5 ( r + 5 ) + 3

distribute 5

G ( r + 5 ) = 5r + 25 + 3

combine like terms

G ( r + 5 ) = 5r + 28

WILL GIVE BRAINLIEST AND 50 POINTS!
Determine the more basic function that has been shifted, reflected, stretched, or compressed.
m(x)= 2√x-5 -2

Answers

Answer: The function M if derived from the integral of the derivative of the function. This is valid by the fundamental theorem of calculus.

M(x)=43x32−7x+C

Step-by-step explanation: not 100 about this but it's what I got

Find the measure of the missing angle using the exterior angle sum theorm.

Answers

Answer:

85°

Step-by-step explanation:

The exterior angle of a triangle is=sum of the opposite interior angles

So

? °=45°+40°

What is the slope of the line? What is the y-intercept of the line? y = -3x + 4

Answers

Answer:

slope= -3/1

y-intercept= 4

Answer:

m = -3

y intercept = 4

Step-by-step explanation:

The given equation of the line is ,

[tex]\implies y = -3x+4[/tex]

We know that the Standard equation of Slope Intercept Form of the line is,

[tex]\implies y = mx + c[/tex]

Where ,

m is slope c is y intercept

On comparing to the Standard form of the line we get ,

[tex]\implies Slope = -3 [/tex]

[tex]\implies y - intercept= 4 [/tex]

In which section of the number line is 32−−√?

Answers

where's the number line?

maybe u can attach it at the comments:)

Answer:

Section B

Step-by-step explanation:

A number ending in ___ is never a perfect square. ​

Answers

Answer:

2, 3, 7 or 8

Step-by-step explanation:

someone help me for this algebra task please

Answers

Answer:

200

Step-by-step explanation:

Substitute 15 for y

[tex] \frac{1}{5} x - \frac{2}{3} (15) = 30[/tex]

[tex] \frac{1}{5} x - 10 = 30[/tex]

[tex] \frac{1}{5} x = 40[/tex]

[tex]x = 200[/tex]

Twenty of the 50 digital video recorders (DVRS) in an inventory are known to be detective What is the probability that a randomly selected item is defective?​

Answers

Answer:

3/5

Step-by-step explanation:

Remember that odds for plus odds against must be 1. Therefore, 20 out of 50 of the DVRs are defective, so 30 out of 50 would work.

30/50

simplifies into 3/5 so that would be the probability of you picking out a DVR that works, and the probability of picking one that does not work would be the leftover 2/5 . Or, if you're answering with a percent, you would have a 60% chance of picking a working DVR.

I hope this helps, I know the explanation isn't really clear, but I can't really think of another way to explain it.

What is 6.273 rounded to the nearest thousandths?

Answers

Answer:

6.270

Step-by-step explanation:

3 is below 5 so you just turn it into a 0. If it was 5 or above you would just add a number to the 7.

if f(x) = 2x/7 +4, which of the following is the inverse of f(x)?

Answers

Answer:

The answer is C

Step-by-step explanation:

Switch x and y

[tex]x = \frac{2y}{7} + 4[/tex]

Solve for y

[tex]y = \frac{7(x - 4)}{2} [/tex]

3 questions only pls help gota finish tofday

Answers

5a) 3x3x13

b) 3x7x7

c)11x5x2x2

d)3x3x3x5x2

6a)2x2x2x131

7a)5x5x2x2

b) 3x2x17

c)5x2x2x2x2x2x2

d)5x5x5x3

e) 2x2x103

f)5x3x137

g) 2x2x2x2x2x2x2x2x2

h)5x5x7x7x3

Identify a horizontal or vertical stretch or compression of the function by observing the equation of the function .

Answers

Bdbxbcjncnxndnsnns Identify a horizontal or vertical stretch or compression of the function by observing the equation of the function .hdbdbebqb

On a coordinate plane, a line goes through (negative 3, negative 4) and (3, 0).
What are the necessary criteria for a line to be perpendicular to the given line and have the same y-intercept?

The slope is Negative three-halves and contains the point (0, 2).
The slope is Negative two-thirds and contains the point (0, −2).
The slope is Three-halves and contains the point (0, 2).
The slope is Negative three-halves and contains the point (0, −2

Answers

Answer:

The slope is Negative three-halves and contains the point (0, 2).

Step-by-step explanation:

(-3,-4)(3,0)

M= -4/-6 = 2/3

⊥M = -3/2

0 = 2/3(3) + B

B=2

In In 5x + In In (x - 1) = 2

Answers

Answer:

exact form: x=-1/2

decimal form: x=-0.5

sue has 18 pieces of candy
tony has 18 pieces of candy
sue then gives some to tony
sue then eats five of hers
tony eats half of his
write the expressions for the number of pieces candy sue and tony now have?

Answers

Answer:

Sue candy = 13 - x

Tony candy = 9 + 1/2x

Step-by-step explanation:

Sue candy = 18

Tony candy = 18

Let x = some candy gives to tony

Sue candy = 18 - x

Tony candy = 18 + x

sue then eats five of hers

Sue candy = 18 - x - 5

= 13 - x

tony eats half of his

Tony candy = 1/2(18 + x)

= 18/2 + x/2

= 9 + 1/2x

Expressions for the number of pieces candy sue and tony now have:

Sue candy = 13 - x

Tony candy = 9 + 1/2x

what is the length of segment DC? no links.

Answers

Answer:

33 units

Step-by-step explanation:

AB = BC

2x + 7 = 23

2x = 16

x = 8

DC = DA

DC = 4x + 1

DC = 4(8) + 1

DC = 33

Someone please help me ASAP

Answers

Step-by-step explanation:

a vector multiplied by a scalar is equal to it's image. The expression above gives an equation and after solving, it gives you the image

log8-log4 ÷ log4-log2=





Answers

The answer is log(4)-1

P(x) is a polynomial. here are a few values of p(x).
P(-5) = - 2
P(-3) = 6
P(3) = 7
P(5) = -1
What is the remainder when P(x) is divided by (x+5)?
What is the remainder when P(x) is divided by (x-3)?

Answers

Given:

Values of a polynomial P(x).

[tex]P(-5)=-2[/tex]

[tex]P(-3)=6[/tex]

[tex]P(3)=7[/tex]

[tex]P(5)=-1[/tex]

To find:

The remainder when P(x) is divided by (x+5).

The remainder when P(x) is divided by (x-3).

Solution:

If a polynomial P(x) is divided by (x-a), then the remainder is P(a).

If the polynomial P(x) is divided by (x+5), then the remainder is P(-5).

[tex]P(-5)=-2[/tex]

Therefore, the remainder is -2 when P(x) is divided by (x+5).

If the polynomial P(x) is divided by (x-3), then the remainder is P(3).

[tex]P(3)=7[/tex]

Therefore, the remainder is 7 when P(x) is divided by (x-3).

Drag the operator to the correct location on the image.
Which operation results in a binomial?

Answers

The correct answer is to drag The Plus sign (+)

What is an Operator?

This has to do with the use of symbols to denote mathematical equations such as addition, subtraction, etc.

Hence, we can see that the correct position to put the operator on the image to result in a binomial is to drag the plus sign (+) so that the equations can be solved,.

Read more about operators here:

https://brainly.com/question/25974538

#SPJ2

Answer:.

Step-by-step explanation:

Which point is in the solution set of this system inequalities?

A. (0,0)

B. None of these

C. (5,1)

D. (3,7)

Answers

Answer:

B

Step-by-step explanation:

To find which ordered pairs are solutions to the inequalities we can simply plug in the x and y values of the ordered pairs into the inequalities and if the equation is true for both inequalities then the ordered pair is a solution to the inequalities.

For (0,0)

x = 0

y = 0

y > x + 5

Substitute 0 for y and x

0 > 0 + 5

Simplify right side

0 > 5

The inequality is not true as 5 is greater than 0, not less than. So immediately we can eliminate answer choice A.

For (5,1).

x = 5

y = 1

y > x + 5

Substitute 5 for x and 1 for y

1 > 5 + 5

Simplify right side

1 > 10

Again, the equation is not true as 1 is not greater than 10. This means that c cannot be the answer

For (3,7)

x = 3

y = 7

y > x + 5

Substitute 3 for x and y for 7

7 > 3 + 5

Simplify right side

7 > 8

7 is not greater than 8 meaning that (3,7) cannot be a solution to the inequalities

None of the ordered pairs created true equations hence the answer is B

t=29pi/6
1. find the reference number
2. find the point on the unit circle
3. 6 trig functiond

Answers

the correct answer is C....I hope

Please help. I don’t understand I rlly appreciate it if you help!

Answers

Hi there!

We know that the angles ∠B and ∠A are supplementary because they are both interior angles. Therefore:

180 = ∠A + ∠B

Rewrite:

180 = (6x - 48) + (4x + 38)

Combine like terms:

180 = 10x - 10

Solve for x:

190 = 10x

x = 19

Solve for ∠B by plugging in this value of x:

∠B = 4(19) + 38

∠B = 76 + 38

∠B = 114

find the volume of the rectangular prism. plz answer this lol

Answers

Answer:

.....how when the dimensions are not even clear lol

Answer:

48 cm³

Step-by-step explanation:

the volume of a rectangular prism= length × breadth × height

= 8× 3 × 2

= 48 cm³

One leg of a right triangle is 7 inches longer than the other leg, and the hypotenuse is 35 inches. Find the lengths of the legs of the triangle.

Answers

Answer: 21, 28

Step-by-step explanation:

Side #1 = xSide #2 = x + 7Hypotenuse = 35

Use the Pythagorean Theorem [tex]a^{2}+b^{2}=c^{2}[/tex]:

a = xb = x + 7c = 35

Substitute in the values & solve:

[tex]x^{2}+(x+7)^{2}=35^{2}\\x^{2}+x^{2}+14x+49=1225\\2x^{2}+14x+49-1225=0\\2x^{2}+14x-1176=0\\2(x^{2}+7x-588)=0\\2(x + 28)(x - 21)=0\\x_{1}=-28, x_{2}=21[/tex]

-28 is not a possible solution since you can't have negative inches...

a = x = 21b = x + 7 = 21 + 7 = 28c = 35

one more question
(-8)+___=-17

Answers

Answer:

-9

Step-by-step explanation:

Answer:

-9

Step-by-step explanation:

-8 +___= - 17

___=-17 +8

___=-9

=-9

help help help pls :)

Answers

Answer:

[tex]opposite\approx 70.02[/tex]

Step-by-step explanation:

The triangle in the given problem is a right triangle, as the tower forms a right angle with the ground. This means that one can use the right angle trigonometric ratios to solve this problem. The right angle trigonometric ratios are as follows;

[tex]sin(\theta)=\frac{opposite}{hypotenuse}\\\\cos(\theta)=\frac{adjacent}{hypotenuse}\\\\tan(\theta)=\frac{opposite}{adjacent}[/tex]

Please note that the names ([tex]opposite[/tex]) and ([tex]adjacent[/tex]) are subjective and change depending on the angle one uses in the ratio. However the name ([tex]hypotenuse[/tex]) refers to the side opposite the right angle, and thus it doesn't change depending on the reference angle.

In this problem, one is given an angle with the measure of (35) degrees, and the length of the side adjacent to this angle. One is asked to find the length of the side opposite the (35) degree angle. To achieve this, one can use the tangent ([tex]tan[/tex]) ratio.

[tex]tan(\theta)=\frac{opposite}{adjacent}[/tex]

Substitute,

[tex]tan(35)=\frac{opposite}{100}[/tex]

Inverse operations,

[tex]tan(35)=\frac{opposite}{100}[/tex]

[tex]100(tan(35))=opposite[/tex]

Simplify,

[tex]100(tan(35))=opposite[/tex]

[tex]70.02\approx opposite[/tex]

plz help me with this math and also explain

Answers

Step-by-step explanation:

[1]

SI = $250Rate (R) = 12[tex] \sf \dfrac{1}{2}[/tex] %Time (t) = 4 years

[tex]\longrightarrow \tt { SI = \dfrac{PRT}{100} } \\ [/tex]

[tex]\longrightarrow \tt { 250 = \dfrac{P \times 12\cfrac{1}{2} \times 4}{100} } \\ [/tex]

[tex]\longrightarrow \tt { 250 = \dfrac{P \times \cfrac{25}{2} \times 4}{100} } \\ [/tex]

[tex]\longrightarrow \tt { 250 = \dfrac{P \times 25 \times 2}{100} } \\ [/tex]

[tex]\longrightarrow \tt { 250 = \dfrac{P \times 50}{100} } \\ [/tex]

[tex]\longrightarrow \tt { 250 \times 100 = P \times 50} \\ [/tex]

[tex]\longrightarrow \tt { 25000 = P \times 50} \\ [/tex]

[tex]\longrightarrow \tt { \dfrac{25000}{50} = P } \\ [/tex]

[tex]\longrightarrow \underline{\boxed{ \green{ \tt { \$ \; 500 = P }}}} \\ [/tex]

Therefore principal is $500.

__________________

[2]

2/7 of the balls are red.3/5 of the balls are blue.Rest are yellow.Number of yellow balls = 36

Let the total number of balls be x.

→ Red balls + Blue balls + Yellow balls = Total number of balls

[tex]\longrightarrow \tt{ \dfrac{2}{7}x + \dfrac{3}{5}x + 36 = x} \\ [/tex]

[tex]\longrightarrow \tt{ \dfrac{10x + 21x + 1260}{35} = x} \\ [/tex]

[tex]\longrightarrow \tt{ \dfrac{31x + 1260}{35} = x} \\ [/tex]

[tex]\longrightarrow \tt{ 31x + 1260= 35x} \\ [/tex]

[tex]\longrightarrow \tt{ 1260= 35x-31x} \\ [/tex]

[tex]\longrightarrow \tt{ 1260= 4x} \\ [/tex]

[tex]\longrightarrow \tt{ \dfrac{1260 }{4}= x} \\ [/tex]

[tex]\longrightarrow \underline{\boxed{ \tt { 315 = x }}} \\ [/tex]

Total number of balls is 315.

A/Q,

3/5 of the balls are blue.

[tex]\longrightarrow \tt{ Balls_{(Blue)} =\dfrac{3 }{5}x} \\ [/tex]

[tex]\longrightarrow \tt{ Balls_{(Blue)} =\dfrac{3 }{5}(315)} \\ [/tex]

[tex]\longrightarrow \tt{ Balls_{(Blue)} = 3(63)} \\ [/tex]

[tex]\longrightarrow \underline{\boxed{ \green {\tt { Balls_{(Blue)} = 189 }}}} \\ [/tex]

Other Questions
22.What tone does this passage convey?Adam ripped up the letter and threw it on the ground. According to the letter, he had not been accepted as a team member. He stomped his foot and yelled, Arg!satiricalpessimisticangryhumorous Ngun lc ca nn kinh t bao gm It took Sarah 4 days to write a paper, she wrote 12 pages on day 1, 15 pages on day 2 and 9 pages on day 3. If she wrote 12 pages per day , how many pages did she write on the fourth day ? soy guapo?si dices que si te doy una galletita Which verb form correctly completes the sentence mis hermanos ___ en un restaurante durante dos meses. n a certain exam of grade ten, 75% students got high score in mathematics, 65%students got high score in English. If 6% of them did not get high score in both mathematics and English, then calculate: i. the percent of students who got high score in both the subjects. ii. the total number of students who got high score either in mathematics or in English if 300 students had attended the exam. I have this math problem I can't solve."Evelyn and Meredith decided to kayak 1 mile up and then back in the Humboldt channel.The rate of the water flowing in the channel was 2 miles per hour. The total time it took them to kayak up and back was 3 hours and 40 minutes. Assuming they were padding their double kayak at a fairly consistent rate, find the rate Evelyn and Meredith were paddling.Step 2 - Draw a picture to model the problem.Step 3 - Label variables and create a table.Step 4 - Write an equation to model the problem.Step 5 - Solve the equation. Provide supporting work and detail.Step 6 - Explain the results."If you can help please do. How can one estimate a car annual fuel expense The dopamine hypothesis of schiziphrenia is based on the theory that: A. Symptoms (mostly positive) of schizophrenia are related to excess dopamins in the brain B. Symptoms (mostly negative) of schizophrenia are related to excess dopamine in the brain C. People with schizophrenia are born with fewer dopamine receptors in the brain D. Lower levels of dopamine contribute to both positive and negative psychotic symptoms (equally) Work out the area of the shape,show working out help me and I think I did the sides wrong What transformation was not done to the linear parent function, f(x) = x, toget the function g(x) = } (x + 5) + 7?A. Reflected over the x-axisB. Vertically compressed by a factor of 2O c. Shifted right 5 unitsD. Shifted up 7 units 1. Explain how your scores compare to the Healthy Fitness Zone Standards. Based on this comparison, what should you do next A person from Quebec is travelling in France. When someone ask him what the weather is like he says, "Il fait frette" to mean, "It's cold." However, the French person is confused. Why? Quebec's language is mainly English, with some French words. This is a qubcois expression only used in Canada. This is a Cajun expression This is part of the Quebec Carnival theme song Question 3 of 10Which statement describes the law of conservation of energy?A. Air resistance has no effect on the energy of a system.B. Energy cannot be created or destroyed.C. The total energy in a system can only increase.D. Energy cannot change forms.SUBMIT solve the quadratic equation x+x-2 Which expression can be used to determine 50% of 42? 42-2 ,422,4210,42-10 SECTION BAnswer ALL questions. Write your answers in the spaces provided.1 Data set A has a median value of 3.1Here is data set B.14-928-38-13-2(a) Write a statement to compare the median values of the two sets of data.(2) in heating a kettle of water on an electric stove, 3.3410^3 J of thermal energy was provided by the element of the stove. yet, the water in the kettle gained only 5.9510^2 J of thermal energy. determine the percent efficiency of the electrical element in heating the kettle of water Capacitance (C) is measured in farads, where 1 farad is equivalent to 1 coulomb per volt. Resistance (R) is measured in ohms, and 1 ohm is equal to 1 volt per ampre. The coulomb is defined as the amount of charge passing when a current of 1 ampre flows for 1 second. What is the unit of RC expressed in its simplest form? alvin is 5 years older than elga. the sum of their age is 85. what is elga age